MBE - Tort Law

¡Supera tus tareas y exámenes ahora con Quizwiz!

How is liability apportioned in a P70%/D30% liability case in a pure modified comparative fault jurisdiction?

. In a modified (i.e., partial) comparative negligence jurisdiction, when there is only one defendant, a plaintiff may recover from the defendant to the extent of the defendant's fault, but the plaintiff is precluded from recovering if his fault exceeds the defendant's fault. Because the plaintiff here was 70% at fault, he is liable for 70% of the defendant's damages (i.e., $700). And, because the plaintiff was more at fault than the defendant, the plaintiff cannot recover anything from the defendant for damages to his own car. Consequently, the defendant is entitled to recover $700 from the plaintiff, making answer choices A, B, and D incorrect.

To what class of persons is the "Extreme" and outrageous element of IIED owed?

A defendant is liable for intentionally or recklessly acting with extreme and outrageous conduct that causes the plaintiff severe emotional distress. Conduct is extreme and outrageous if it exceeds the possible limits of human decency, so as to be entirely intolerable in a civilized society. Courts are more likely to find a defendant's abusive language and conduct to be extreme and outrageous if the plaintiff is a member of a group with a known heightened sensitivity (e.g., a pregnant woman). Here, the wife is a member of a group with a known heightened sensitivity because she is pregnant, so a court would be more likely to find her husband's conduct to be extreme and outrageous. In addition, the husband fed his wife meat products on a daily basis for about seven months, and this amount of long-term and repetitive conduct in conjunction with the fact that she is pregnant would further support a finding that the husband's behavior was extreme and outrageous.

Rule statement: trespass to chattels

A defendant is liable for trespass to chattels if he intentionally interferes with the plaintiff's right of possession by either dispossessing the plaintiff of the chattel or using or intermeddling with the plaintiff's chattel. In the case of dispossession, a plaintiff must prove damages by either the actual damages caused by the interference or the loss of use. In the case of use or intermeddling, the plaintiff must show actual damages. In this case, the brother cannot prove he suffered any damages. Accordingly, his claim will likely fail.

What does the duty to warn entail?

A failure-to-warn theory of recovery is the strongest argument. To prevail, a plaintiff must prove that the manufacturer failed to provide an adequate warning related to the risks of using the product. A failure to warn defect exists if there were foreseeable risks of harm, not obvious to an ordinary user of the product, which risks could have been reduced or avoided by providing reasonable instructions or warnings. The failure to include the instructions or warnings renders the product not reasonably safe. Here, there was a foreseeable risk that a consumer would leave the corrosive paint-removing chemical on a surface for too long and cause damage to the surface. The fact that the chemical could damage a hardwood floor if left on the floor for too long may not be obvious to an ordinary user of the product, and the manufacturer could have easily put a warning on the product stating that it should not be left on any surface for more than a certain amount of time.

What standard of care is owed to a trespasser in attractive nuisance?

A land possessor owes an invitee the duty of reasonable care, including the duty to use reasonable care to inspect the property, discover unreasonably dangerous conditions, and protect the invitee from them. Here, the child was an invitee, and Jumping Land owed him a duty of reasonable care to inspect the facility and protect the child from unreasonably dangerous conditions. Jumping Land satisfied this duty as evidenced by the daily safety inspections. However, in this case, the child was a trespasser who exceeded the scope of the invitation at the time of the injury because his father opened the childproof gate for him. The duty of reasonable care does not extend to trespassers. However, because the brightly painted air machines would likely qualify as an attractive nuisance, Jumping Land had a duty to exercise reasonable care in protecting the child from unreasonably dangerous conditions. It satisfied this duty as evidenced by the childproof gates and warning signs surrounding the air machines. Answer choice A is incorrect because the air machines are not abnormally dangerous. Abnormally dangerous means that an activity creates a foreseeable and highly significant risk of physical harm even when reasonable care is exercised, and the activity is not commonly engaged in. In this case, the use of the air machines does not create a foreseeable and highly significant risk of physical harm even when reasonable care is exercised. By performing daily inspections and gating the air machines, as well as posting warning signs, Jumping Land exercised reasonable care which minimized any foreseeable risk of physical harm. Thus, the air machines do not fit the definition of abnormally dangerous.

What duty of care does a business owe to invitees?

A land possessor owes an invitee the duty of reasonable care, including the duty to use reasonable care to inspect the property, discover unreasonably dangerous conditions, and protect the invitee from those conditions. Here, the bar, as a business premise, had a duty to protect the woman, a business visitor, from the slippery floor. Although the bar occasionally scattered sawdust on part of the floor to absorb the excess water, the use of sawdust only marginally improved the safety conditions. Thus, the occasional use of sawdust likely does not satisfy the bar's duty to exercise reasonable care in protecting patrons from the wet floor. Therefore, the woman will likely succeed.

What duty of care is owed to licensees?

A licensee is someone who enters the land of another with the express or implied permission of the land possessor. The land possessor has a duty to either correct or warn a licensee of concealed dangers that are known to the land possessor. The land possessor must also refrain from willful or wanton misconduct. Here, the teacher is a licensee because the owner permitted the teacher and schoolchildren to enter the land to pick their own pumpkins. The owner's duty, therefore, was to not willfully or wantonly cause injury to the schoolchildren and the teacher and to warn them of any known dangers. The owner did not willfully or wantonly cause the teacher's injury. Furthermore, the owner warned the schoolchildren and the teacher by posting a clearly visible sign regarding the presence of reptiles and rodents in the patch. Thus, the owner will not be held liable. Answer choice A is incorrect because strict liability for animals refers to when the owner knowingly keeps the animal on his property (e.g., as a pet).

Do commercial suppliers or. Manufacturers have a duty to warn of obvious dangers?

A manufacturer has no obligation to warn against obvious dangers. There appears to be nothing unique to the furnace that would add to the ordinary dangers of working on a ladder. Answer choice B is incorrect. Furnaces should be built for ordinary use, including routine maintenance by homeowners. In addition, there is no indication that a professional would be more adept than the homeowner in maintaining his balance on the ladder; maintaining one's balance on a six-foot stepladder is not a professional technical skill. The homeowner will likely not prevail, but the reason is that the manufacturer had no duty to warn of the obvious risks of working on a ladder.

When is pure economic loss available?

A negligence plaintiff must prove actual injury. A plaintiff who suffers only economic loss without any related personal injury or property damage cannot recover such loss through a negligence action. In this case, the business suffered only economic losses, and thus may not recover. Answer choice B is incorrect because the business may not recover from either the driver or his employer. Moreover, an employer is liable for the negligent actions of its employee committed in the scope of employment. Answer choice C is incorrect because the business suffered only economic loss, not actual damages. Answer choice D is incorrect because, even if the driver owed a duty of care to the business, the business cannot satisfy the elements of a negligence action because it cannot prove damages

Define honest but mistaken belief., RS

A person may use reasonable force to defend against an offensive contact or bodily harm that he reasonably believes is about to be intentionally inflicted upon him. A person's mistaken belief that he is in danger, so long as it is a reasonable mistake, does not invalidate the defense. Under the circumstances, it is not probably not reasonable to assume that a man who jogged across a busy street and smiled at the woman after she had dropped her groceries planned to attack her. Answer choice A is incorrect because the majority of jurisdictions do not require an attempt to retreat before the use of force. Answer choice B is incorrect. A person's particular physical characteristics, such as blindness or deafness, are taken into account when determining the reasonableness of their actions. However, even taking the elderly woman's deafness into account here does not make her reaction reasonable; further, even a person on average hearing ability may not have been able to hear the man across the busy street. Therefore, even considering this physical characteristic, the woman's belief was not reasonable. Answer choice D is incorrect. When a defendant was the initial aggressor because she was responding to an honest and reasonable belief that an offensive contact or bodily harm was about to be inflicted upon her, she could claim self-defense. Therefore, even though the elderly woman attacked the man first, she could have claimed self-defense if her belief was reasonable. There is no requirement of a threat of deadly force.

Which duties can be delegated to an IC?

A person who hires an independent contractor is not generally vicariously liable for the torts committed by an independent contractor. However, a person who hires an independent contractor remains vicariously liable for the breach of non-delegable duties, such as the duty of a shopkeeper to keep premises that are open to the public in a reasonably safe condition. In this case, that duty was breached and consequently the storeowner is vicariously liable for the negligence of the employee of the flooring company, even though the flooring company was an independent contractor.

When is battery from behind possible?

A plaintiff can recover for negligent infliction of emotional distress from a defendant if the plaintiff demonstrates that he was within the "zone of danger" of a threatened physical impact, that he feared for his own safety because of the defendant's negligence, and the threat of physical impact caused emotional distress. Here, the kayaker was within the zone of danger of the threatened impact of the wake intentionally caused by the boat driver's erratic driving. This threat of physical impact also caused the kayaker emotional distress for which he had to see a therapist. These facts satisfy the elements of a claim for negligent infliction of emotional distress. The boat driver is also liable for battery. A defendant is liable to a plaintiff for battery when he causes a harmful or offensive contact with the person of another, and acts with the intent to cause such contact or the apprehension of such contact. Here, the boat driver intended a harmful and offensive contact against the water skier when he tried to make him fall into the water. However, under the doctrine of transferred intent, the boat driver is also liable to the kayaker for battery because he intended to commit a battery against the water skier and inadvertently caused a harmful contact to the kayaker.

What is the standard of care owed by a professional?

A professional person, including a lawyer, is expected to exhibit the same skill, knowledge, and care as another practitioner in the same community. A public defender is a professional, so in deciding whether she was negligent, the jury should apply this standard of care.

What duty of care is owed by a common carrier?

A special relationship exists between common carriers, such as operators of cruise ships, and their passengers. As such, a common carrier has a duty to use the highest level of care consistent with the practical operation of its business. Accordingly, the cruise ship should be liable in negligence for its breach of its duty of care to the friend for failing to fix the broken lock. Answer choice B is incorrect as it implies a strict liability standard for common carriers. Answer choice C is incorrect. Proximate cause requires that the plaintiff suffer a foreseeable harm that is not too remote and is within the risk created by the defendant's conduct. Here, the friend could have accidentally locked himself in the cabin and been unable to escape due to the broken lock, regardless of the actions of the man. Therefore, this harm is not too remote and the cruise ship can be held liable. Answer choice D is incorrect. Although there is typically no duty to rescue another in peril, a common carrier, such as a cruise ship, has a higher standard of care with respect to its passengers, and will be liable for failing to repair the lock that trapped the friend.

What. Damages. Are available for trespass?

A trespass to land occurs when one's intentional act causes a physical invasion of another's land. A trespasser need only have the intent to enter the land (or to cause a physical invasion), not the intent to commit a wrongful trespass. Here, the wife committed trespass to land because she intentionally entered the neighbor's backyard. The privilege of necessity is available to a person who enters onto the land of another in order to prevent injury that is substantially more serious than the invasion or interference itself. Private necessity is a qualified privilege to protect an interest of the defendant or a limited number of people. The privilege applies if the interference was reasonably necessary to prevent a threatened injury from nature or another force not connected with the plaintiff. The property owner is entitled to recover actual damages, but he cannot recover nominal or punitive damages nor can he use force to eject the defendant. In this case, the privilege of private necessity is available to the wife because crushing the tomato plants and contaminating the neighbor's pool is less serious than the danger of her home burning down, and it was reasonably necessary to use the neighbor's pool to put out the fire. Despite the privilege, the neighbor is still entitled to recover actual damages. However, he cannot recover nominal or punitive damages. For this reason, answer choices A, C and D are incorrect.

PLC may be liable to Peter for both an abnormally dangerous activity, as well as an attractive nuisance. How do we organize our headings?

A. Strict Liability B. Negligence (Duty - class (children) Breach - C D)

Attractive nuisance mnemonic + elements

AHA! BR! Artificial/Attract Harm Appreciate Balance Reasonable Care

What are the two types of causation??**

Actual cause (cause in fact) and proximate cause

What harm is a negligent medical professional liable for following an improperly conducted surgery?

Although the plaintiff's failure to promptly seek medical attention aggravated his injuries, it was the aggravated injury that the defendant agreed to treat. Consequently, the surgeon is not liable for such injuries, but only for the harm caused by the surgeon's negligent treatment of those injuries. (Note: When the plaintiff's initial harm is inflicted by the defendant, rather than by the plaintiff or an independent third party, the plaintiff's failure to promptly seek medical care can constitute a failure to mitigate damages, which can be taken into account in determining the amount of damages to which the plaintiff is entitled.) Answer choice A is incorrect because the loss of future earnings that is attributable to harm that is caused by the defendant can be recovered by a plaintiff, if such loss can be established with reasonable certainty. Answer choice C is incorrect because damages for emotional distress are recoverable in a negligence action when the plaintiff has suffered a physical injury even though the emotional distress itself has not been manifested through a physical injury. Answer choice D is incorrect because a failure by the plaintiff to mitigate his damages can be taken into account in determining the amount of the plaintiff's damages.

When does self defence become a battery?

Answer choice A is correct because the deliveryman committed a battery. His intentional tripping of the freshman caused the freshman's injuries. Answer choice B is incorrect because, although the deliveryman did interpose himself into the students' affair, an intermeddler may nevertheless escape liability when the defense of others applies. Answer choice C is incorrect because, although the deliveryman used reasonable force in tripping the freshman, the deliveryman could not avail himself of the defense of others. The deliveryman could not step into the shoes of the senior because the senior, by chasing the freshman, was not acting in self-defense but instead was retaliating. In addition, the deliveryman could not assert that he had a reasonable belief that the senior was acting in self-defense, because the senior was the pursuer rather than the pursued. Answer choice D is incorrect because, although the freshman was the initial aggressor by continuing to roughhouse after the senior withdrew his consent, the senior became the aggressor by chasing the freshman, thereby acting in retaliation rather than self-defense.

What duty is owed to known trespassers under the third restatement?

Answer choice C is correct. Landowners owe a duty toward discovered or anticipated trespassers only to warn or protect them from concealed dangerous artificial conditions. But here, the owner's knowledge of the homeless population in the building and his acquiescence in their presence converts them to licensees, as they essentially entered his building with his implied permission, so he must do more than warn. Therefore, according to the Third Restatement, the owner should have exercised reasonable care to prevent harm to the entrants posed by artificial conditions (such as the uneven floors). Here, the owner should have exercised reasonable care to ensure the safety of the building, as he knew and impliedly approved of the entrants being in the building. Answer choice A is incorrect because although the risk of harm may not have been so severe, the owner still should have exercised reasonable care to protect against the harm associated with the uneven floors. Answer choice B is incorrect because these individuals were on the premises with the implied permission of the owner, making them licensees, not trespassers. As discussed above, landowners have a duty to protect licensees from conditions that posed a risk of harm. Answer choice D is incorrect because the owner did not have to ban the people from entering the building, provided he took reasonable care to ensure the building was safe. Note, too, that denying permission to enter the building would not absolve the landowner of all liability, as some duties are owed to trespassers, particularly discovered ones.

What. Is judgment as a matter of law?

Answer choice C is correct. Negligence occurs when a defendant owes a duty of care to the plaintiff, breaches that duty of care, and the breach of the duty of care actually and legally caused the plaintiff's harm, resulting in damages. Here, the court should not grant the motion for judgment as a matter of law, as it is possible that the jury could find that the sisters breached their duty of reasonable care to the elderly woman, who was a foreseeable plaintiff. The sisters were not accustomed to lifting heavy furniture, they wore high heels while moving the couch, and they had shared a bottle of wine prior to doing so. It was foreseeable that they could lose control of the couch, which could result in injury to someone climbing the stairs behind them. Answer choice A is incorrect. The elderly woman may have assumed the risk of being harmed by the falling couch by following behind the sisters. However, even if assumption of the risk was applicable here, it would not prevent her from recovering damages in a pure comparative negligence jurisdiction (applies when no other type of jurisdiction is specified), although the elderly woman's recovery would be reduced. Answer choice B is incorrect. If the sisters had not lost control of the couch, the elderly woman would not have tried to jump out of the way. The injury the elderly woman received is sufficiently related to the sisters' act that they may be liable, and is not so remote that the sisters cannot be found to be the legal cause of the injury. Answer choice D is incorrect because the elderly woman brought an action for negligence. Therefore, the elderly woman's apprehension of imminent harmful contact is not relevant and is not determinative of the success of the motion for judgment as a matter of law.

What specific conditions must a landowner warn a trespasser of?

As a discovered trespasser, the golf course owed a duty to warn or protect the man from concealed, dangerous, artificial conditions, such as the live electrical wires. The manager had reason to believe that the man was unaware of the concealed electrical wires, the dangerous artificial condition, and failed to warn him.

What liability applies to an employer for their employee's intentional tort?

As a general rule, an employer is liable for the tortious conduct of an employee that is within the scope of employment. Generally, an employer is not liable for the intentional torts of an employee. However, when force is inherent in the employee's work, the employer may be responsible for injuries the employee inflicts in the course of his work. Here, force was inherent in the bodyguard's job to protect the singer. The bodyguard acted within the scope of his employment when he punched the man in the face because, when the man suddenly lurched towards the singer, the bodyguard used reasonable force to protect the singer. Answer choice A is incorrect because an employer can be liable for the intentional torts of an employee when force is inherent in the employee's work and the tort was within the scope of his employment. Answer choice B is incorrect. Careful instructions directed to the employee do not insulate the employer from liability—even when the employee acts counter to the instructions—if the employee is acting within the scope of employment. Here, the singer told the bodyguard to use reasonable force against a fan only if the fan came within a one-foot radius of the singer. Although the bodyguard acted counter to these instructions, he was still acting within the scope of his employment under his contract because the man posed a threat to her safety when he lurched towards her. Answer choice C is incorrect because an employer is not always responsible for an employee's tortious conduct when force is inherent to the employee's job. The employee's tortious conduct must fall within the scope of employment.

To whom is a duty owed under both the majority and minority view?

CARDOZO (ZOne of foreseeable harm - class of persons foreseeably injured by that harm) or ANDREWS (foreseeable harm - everyone) The majority rule (the "Cardozo view") is that a duty of care is owed to the plaintiff only if she is a member of the class of persons who might be foreseeably harmed as a result of the defendant's negligent conduct. This means a defendant is liable for negligence only to those plaintiffs who are "within the zone of foreseeable harm." The minority view (the "Andrews view") states that if the defendant can foresee harm to anyone as a result of his negligence, then a duty is owed to everyone (foreseeable or not) injured as a result of his breach.

What should always be the first defence to negligence? MNEMONIC for all defences.*

CC (out IN out) - AR - BAR Contributory/Comparative Negligence (Contributory - OUT - contribute = complete) Comparative - PU CO (reduces) MOD=MIDI (NO if 50%+) Contributory/comparative negligence occurs when a plaintiff fails to exercise reasonable care for his own safety and thereby contributes to his own injury. At common law, contributory negligence is a complete bar to recovery. In jurisdictions that have adopted the doctrine of pure comparative negligence, a plaintiff's contributory negligence is not a complete bar to recovery. Rather, the plaintiff's damages are reduced by the proportion that his fault bears to the total harm. In a partial comparative negligence jurisdiction, the plaintiff's recovery is barred if he was more than 50% at fault. Here, Peter was likely negligent for chasing his bird into the fenced-in area, and his injuries were due in part to his own negligence. Peter, as a reasonable 12-year-old, could read and understand the warning signs, and he likely appreciated the danger posed by the substation surrounded by a six-foot tall fence topped with barbed wire. Peter was negligent in failing to heed these warnings; thus, Father's recovery may be barred or reduced. In a contributory negligence state, Father will not recover at all because Peter's negligence is a complete bar to recovery. In a comparative negligence state, Father's recovery will be reduced by the proportion of Peter's negligence (pure comparative jurisdiction) or barred if Peter's negligence exceeded 50% (partial comparative negligence).

RS for conversion

Conversion is equivalent to a forced sale of the chattel to the defendant, who is liable for the full value of the chattel at the time of the tort. The tort occurred when the bank refused to relinquish the stock certificate in response to the customer's lawful demand, and at that time the shares were worth $20,000. Answer choice A is incorrect. Someone who refuses to surrender a chattel to another person who is entitled to its immediate possession is liable for conversion even if the one holding the chattel originally came into possession lawfully. The bank is liable to the customer for the value of the shares at the time the bank refused the customer's lawful demand for return of the certificate - $20,000. Answer choices B and C are incorrect because the defendant is liable for the full value of the chattel at the time of the tort - when the bank refused to relinquish the stock certificate in response to the customer's lawful demand, and at that time the shares were worth $20,000.

Rule statement: motion for a directed verdict

DEARTH of EVidence A directed verdict is a decision by a judge that awards judgment to the defendant without the defendant having to put on his own case. A directed verdict is typically awarded when the plaintiff has *failed to offer a prima facie case* and cannot establish all of the requisite elements of the claim, even if the defendant presents no opposition. Here, if a jury found that the manager owed the patron, an invitee, a duty of reasonable care, which he breached by serving him food with nuts in it, and causing him a severe allergic reaction, a directed verdict should not be issued. Answer choice A is incorrect, because as stated above, a judge can award judgment to the defendant without the defendant having to put on his own case. Answer choice C is incorrect. A plaintiff's voluntarily encountering a known, specific risk is an affirmative defense to negligence that can either reduce or in some cases bar recovery; most courts hold that the voluntary encountering must also be unreasonable. Here, the patron did not know about the specific risk of nuts in the chicken dish because the menu explicitly stated that there were no nuts in the dish. The footnote on the menu stating that the same kitchen also handled nuts and animal products does not make it unreasonable for the patron to rely upon the statement in the menu that the chicken dish was free of nuts. Answer choice D is incorrect, because consent is a defense to intentional torts, whereas assumption of the risk applies to negligence actions. Thus, the fact that the patron consented to ordering the chicken dish would not permit the manager to escape liability in a negligence suit.

IIEC Elements

DICK - Damages result from breach Intent to cause breach of K Valid contract Knowledge that contract exists To prove intentional interference with a contract, the regional retailer must prove that (1) a valid contract existed between the regional retailer and the manufacturer, (2) the national retailer knew of the contractual relationship, (3) the national retailer intentionally interfered with the contract, causing a breach, and (4) the breach caused damages to the the regional retailer. The regional retailer need only show a pecuniary loss of the benefits from the contract; there is no requirement that the interference prevent the regional retailer from being profitable or that the loss be substantial. Therefore, the argument that the national retailer did not substantially interfere is a weak argument, and answer choice B is correct. Answer choice A is incorrect because when the interference involves a prospective economic relationship, rather than an existing contractual relationship, the defendant has greater latitude in acting in its own interest, particularly if the plaintiff is a competitor. Therefore, answer choice A would be a strong argument for the national retailer to make. Answer choice C is also a strong argument and therefore incorrect because the relationship between the parties is a factor in determining whether the interference is improper. Generally, a competitor has greater latitude in acting in a manner that interferes with the business relationships of a competitor. Answer choice D is incorrect because a person is generally free to deal or to refuse to deal with another. Thus, this argument is stronger than answer choice B.

When is deadly force in the defence of another permitted?

Defense of another is permitted upon reasonable belief that the defended party would be entitled to use self-defense. The defender may use force that is proportionate to the anticipated harm to the other party. A person may use deadly force only if he has a reasonable belief that force sufficient to cause serious bodily injury or death is about to be intentionally inflicted upon him. Here, the patron's threat to use deadly force against the man was too remote in time to be considered valid grounds for resorting to deadly force. Answer choice A is incorrect because the defender of another may use reasonable force that is proportionate to the anticipated harm to the other party, including deadly force when warranted. Answer choice C is incorrect because the patron's threat to use deadly force was not a threat of the immediate use of such force and therefore did not justify the use of deadly force. Answer choice D is incorrect because, although defense of another is permitted upon reasonable belief that the defended party would be entitled to use self-defense, the force must be proportionate to the anticipated harm to the other party. In this case, the force was not proportionate.

What duty is owed to undiscovered trespassers?

Dora does not have a valid claim against the neighbor for negligence because the neighbor does not owe a duty of care to undiscovered trespassers. A landowner is obligated to refrain from willful, wanton, reckless, or intentional misconduct toward trespassers. Landowners generally owe no duty to undiscovered trespassers; nor do they have a duty to inspect their property for evidence of trespassers. Here, Dora trespassed onto her neighbor's property and was injured as the result of an artificial condition on the neighbor's property, the lumber left on the ground. The neighbor, however, was not aware of Dora's presence. Accordingly, the neighbor did not owe a duty of care to Dora.

What must you always discuss for Negligence?*

Duty (normal or special class) Breach Causation (actual - but for, if substantial factor D still liable - and proximate cause) Damages

What are the requirements to establish strict products liability on the basis of failure to warn?

Foreseeable risk, could Have been avoided, not known to ordinary consumer Here, the woman has brought an action under a strict products liability theory for failure to warn, which involves a manufacturer's failure to provide an adequate warning related to the risks of using the product. A failure to warn defect exists if there were foreseeable risks of harm not obvious to an ordinary user of the product, and those risks could have been reduced or avoided by providing reasonable instructions or warnings. The failure to include the instructions or warnings renders the product not reasonably safe. Here, it would not be obvious to an ordinary user of this skin cream that you would need to use a sunscreen with an SPF of 40 or higher, and there was a foreseeable risk of harm that could have been avoided if the manufacturer had put this information on the skin cream label. Answer choice A is incorrect. It would not be obvious to an ordinary user of the cream that the user would have to use a sunscreen with an SPF of 40 or higher. Even if an ordinary user would know that sunscreen is generally an advisable precaution against burns, an ordinary user would not assume without notice that an SPF of 40 or higher would be required to use a particular skin cream. Answer choice B is incorrect. Under the "learned-intermediary" rule, the manufacturer of a prescription drug typically satisfies its duty to warn the consumer by informing the prescribing physician of problems with the drug rather than informing the patient taking the drug. However, because the manufacturer did not inform the dermatologist about the SPF requirement, the dermatologist's failure to warn the woman does not relieve the manufacturer of liability.

Abnormally dangerous activity*

HUGIV Created a foreseeable and highly significant risk of physical *harm, even when reasonable care* is exercised b. The activity is *uncommonly* engaged in; and c. The courts should consider the *gravity* of the harm to P, the *inappropriateness* of the location, and the *limited value* of the harm to the community. *A prima facie case for strict liability requires (i) an absolute duty to make the plaintiff's person or property safe, (ii) breach, (iii) actual and proximate causation, and (iv) damages.*

What are the requirements for false imprisonment and the shopkeeper's privilege?

ICCC Intends to Confine Causes Conscious Reasonable detention permitted (totality of circumstances) False Imprisonment False imprisonment results when a person acts: i) Intending to confine or restrain another within boundaries fixed by the actor; ii) Those actions directly or indirectly result in such confinement; and iii) The other is conscious of the confinement or is harmed by it. 1. Confined Within Boundaries The plaintiff must be confined within a bounded area in which the plaintiff's freedom of movement in all directions is limited. The bounded area may be large and need not be stationary. An area is not bounded if there is a reasonable means of safe escape. 2. Methods of Confinement The defendant may confine or restrain the plaintiff by the use of physical barriers, physical force, direct or indirect threats (to the plaintiff, a third party, or the plaintiff's property), or by the invalid use of legal authority, duress, or the failure to provide a reasonable means of safe escape. Shopkeeper's Privilege: A shopkeeper's reasonable (in both duration and manner) detention of a suspected shoplifter is not an invalid use of authority and hence is not a false imprisonment. Furthermore, a court may find that the defendant has confined the plaintiff when he has refused to perform a duty to release the plaintiff from an existing confinement or provide a means of escape. Example: If a child accidentally locks herself in a restroom in a restaurant, the restaurant may be liable if it intentionally fails to assist her in unlocking the door to obtain her release. The defendant's use of moral pressure or future threats does not constitute confinement or restraint. The plaintiff is not imprisoned if she submitted willingly to confinement. 3. Time The length of time of the confinement or restraint is immaterial, except as to the determination of the extent of damages. 4. Intent The defendant must act with the purpose of confining the plaintiff or act knowing that the plaintiff's confinement is substantially certain to result. If the confinement is due to the defendant's negligence rather than his intentional acts, then the defendant may be liable under the rules governing negligence but not under the intentional tort of false imprisonment. If the imprisonment occurs by pure accident and involves neither the defendant's intent nor his negligence, then there is no recovery. The doctrine of transferred intent applies to false imprisonment. 5. Damages It is not necessary to prove actual damages (except when the plaintiff is unaware of the confinement). Punitive damages may be imposed in appropriate cases. Shopkeepers Privilege - Apple genius bar is REASONABLE, STEIN SUX The shopkeeper's privilege doctrine, as recognized in most states, allows shopkeepers to prevent suspected shoplifters from leaving the premises as long as the detention is for a reasonable time and effectuated in a reasonable manner. The reasonableness of a detention is based on the totality of the circumstances, and is the province of the fact finder. The facts and circumstances given in the instant problem suggest that the detention of the shopper was reasonable. While she was locked in a room without a reasonable means of escape, the detention continued only for a short time. Thus, the security guard stayed within the bounds of the shopkeeper's privilege doctrine, and a false imprisonment claim will not lie. Answer choices A and B are incorrect because the shopkeeper's privilege provides a complete defense to a false imprisonment claim. Answer choice D is incorrect because it is not supported by the given facts.

Defamation of public official, when?

In a defamation action brought by a candidate for public office, the plaintiff must establish more than mere negligence with regard to the truth or falsity of the allegedly defamatory statement of fact. The plaintiff must establish that the defendant acted with actual malice, that is, that the defendant in fact knew the statement to be false or entertained serious doubts as to the truth of the statement. Here, the candidate cannot establish actual malice on the part of the editor in publishing the statement. Answer choice A is incorrect. The assertion that the candidate used illegal drugs purported to be a statement of fact, not a statement of opinion. Defamation turns on what is conveyed in the statement published by the defendant. The context may influence what is conveyed, but facts may be stated in editorials or advertisements as well as in news reports. In this case, it is true that the candidate will not recover, but the reason is that he cannot show the actual malice required to defame a political candidate because the editor believed the statement about the candidate's drug use to be true. Answer choice C is incorrect. The plaintiff in a defamation action must establish that a statement is defamatory, and accusing someone of a criminal act is indeed "defamatory per se." However, a political candidate, like a political official, must also establish that the defendant acted with actual malice. Here, the candidate cannot establish that essential element of his case. Answer choice D is incorrect because the evidence does not support a finding of actual malice in this case.

When is assumption of risk a full defence?

In general, a duty of care is owed to all foreseeable persons who may foreseeably be injured by the defendant's failure to act as a reasonable person of ordinary prudence under the circumstances. Here, the man had a duty to warn the neighbor of the overheating issue with the edger as the neighbor was going to use it. By failing to do so, the man breach his duty of care to the neighbor. Therefore, he can be held liable for the neighbor's damages. Answer choice A is incorrect. Strict liability generally occurs in three situations: wild animals, dangerous activities, and defective or dangerous products. Here, the only possible claim for strict liability would be based on a defective product due to the issue with the component in the edger. However, the man cannot be held liable under a strict products liability theory because he merely lent the edger to the neighbor and was not a seller. Moreover, even if he had sold the edger to the neighbor, the man would likely be considered a casual seller and therefore could not be held strictly liable. Answer choice C is incorrect. Contributory negligence occurs when a plaintiff fails to exercise reasonable care for her own safety and thereby contributes to her own injury. Here, an argument can be made that the neighbor contributed to his own injury by continuing to use the edger after noticing that it had become hot. However, as pure comparative negligence should be applied on the MBE unless otherwise stated, any negligence on the neighbor's behalf would only reduce the neighbor's recovery; it would not preclude recovery. Answer choice D is incorrect. Although the neighbor knew the edger started to feel hot, he did not know that there was a defect in the edger's component that could cause serious injury. Therefore, the neighbor did not knowingly assume the risk of being burned by the defective edger.

Rule statement: affirmative duty to act

In general, there is no affirmative duty to act. A person who voluntarily aids or rescues another has a duty to act with reasonable ordinary care in the performance of that aid or rescue, and a person who places another in peril is under a duty to exercise reasonable care to prevent further harm by rendering care or aid. Here, the skier did no more than stop and examine the snowboarder. Therefore, she did not assume a duty to protect the snowboarder, and unless she placed him in additional peril, she did not breach any duty to him. Answer choice A is incorrect. The last clear chance doctrine applies in contributory negligence jurisdictions, but pure comparative negligence applies on the MBE unless the facts specify otherwise. Answer choice B is incorrect. When the tortious acts of two or more defendants are each a factual cause of an indivisible injury to the plaintiff, the defendants are jointly and severally liable. Under the doctrine of joint and several liability, each of two or more defendants who is found liable for a single and indivisible harm to the plaintiff is subject to liability to the plaintiff for the entire harm. Thus, even if the first skier establishes that the other skier was a legal and but-for cause of the snowboarder's concussion, this would only help establish the other skier's negligence, rather than rebut the claim against her. Therefore, answer choice C is not the first skier's strongest defense. Answer choice D is incorrect because in a pure comparative negligence jurisdiction, the plaintiff's negligence will reduce, rather than bar his recovery. Therefore, this is not the first skier's strongest defense.

Under a negligence theory, what standard of care is imposed on an unintentionally intoxicated individual?

In most cases, the standard of care imposed is that of a reasonably prudent person under the circumstances, as measured by an objective standard. Intoxicated individuals are held to the same standards as sober individuals unless their intoxication was involuntary. Here, the student's intoxication was involuntary, as she did not know the cookie contained any illicit substance. Therefore, she should not be held to the standard of a sober person. Answer choice A is incorrect because, while a sober person may be a reasonable person, that isn't necessarily the case. Moreover, an intoxicated individual is not held to the same standard as a sober individual when the intoxication is involuntary. Answer choice B is incorrect because intoxication is not considered a physical characteristic. Answer choice D is incorrect because being intoxicated does not necessarily relieve one of liability; the issue is whether the intoxication was voluntary or not.

Is a statute always evidentiary of the standard of care?

In most cases, the standard of care imposed is that of a reasonably prudent person under the circumstances. This standard is an objective one, measured by what a reasonably prudent person would do, rather than whether a particular defendant is acting in good faith or using her best efforts. In most situations, no expert testimony is required to define this standard of care, particularly when the defendant's negligence is so apparent that a layperson can identify it without the assistance of an expert in the field. Therefore, the jury is able to determine the standard of care applicable in this situation without expert testimony because the contractor used rusty brackets, rotting wood, and did not properly tighten the fastening elements on the deck. Answer choice A is incorrect. In some cases, a statute can be conclusive evidence of a standard of care. A violation of these statutes is considered negligence per se. However, the criminal or regulatory statute must impose a penalty for a violation of a duty it creates for this concept to apply. Because these safety codes impose no penalty, they cannot be used to conclusively establish a standard of care. Instead, they are merely evidence of negligence to be considered in the jury's objective determination of the proper standard of care in this case.

What is the minority approach to negligence per se?

In some cases, the standard of care owed by a defendant can be determined by statute. In most jurisdictions, this violation can establish negligence as a matter of law. In a minority of jurisdictions, however, violation of that statute can give rise to a rebuttable presumption that the defendant owed a duty and breached it. This violation is referred to as negligence per se. Here, the pedestrian violated a code that imposed a fine for trespassing on construction sites, which the pedestrian did. Therefore, in a minority jurisdiction such as this one, this violation creates a rebuttable presumption that the pedestrian breached a duty. Accordingly, answer choice A is incorrect. Answer choice C is incorrect. The default rule on the MBE is pure comparative negligence, unless the facts instruct you to apply a different test. In a pure comparative negligence jurisdiction, a plaintiff's contributory negligence is not a complete bar to recovery. Therefore, the fact that the pedestrian may have been negligent as a matter of law would not prevent the pedestrian from recovering damages from the city (if the city was somehow negligent). Answer choice D is incorrect. Even if the worker's act contributed to the cause of the accident, the pedestrian's violation of the code has the legal effect of creating a rebuttable presumption that the pedestrian breached a duty.

How does a finding of vicarious liability impact the ability to sue others for contribution?

Indemnification generally applies when one tortfeasor is vicariously liable for the other's wrongdoing. Here, because the facts provide that the neighbor's judgment relied at least in part on a finding of vicarious liability, the landowner can probably seek indemnity from the landscaper. Answer choice B is incorrect because, based on these facts, it is unclear whether the landscaper breached a duty to the landowner. The court has found that the landowner was negligent when he installed the koi pond, and that he was vicariously liable for negligence committed by the landscaper. However, this finding only means that the landscaper has breached a duty to the neighbor, since the neighbor was the plaintiff in that action. Additionally, the only loss the landowner has suffered is an adverse judgment, and a plaintiff cannot recover economic damage in a negligence action without any related personal or property damage. On these facts, it is not clear that the landowner has suffered any damages that would support a claim of tortious negligence against the landscaper.

What should always come to mind for physician negligene? RS?

Informed consent duty too obtain. Negligence is the commission of an act (or the failure to act), without wrongful intent, that falls below the minimum degree of ordinary care imposed by law to protect others against unreasonable risk of harm. Physicians are under a specific obligation to explain the risks of a medical procedure to a patient in advance of a patient's decision to consent to treatment. Failure to comply with this "informed consent" doctrine constitutes a breach of the physician's duty owed to the patient and is actionable as medical malpractice (medical negligence). By failing to discuss the patient's allergies, and the risk of death from the eggs in the vaccine, the doctor breached his duty to the patient to obtain informed consent. Answer choice B is incorrect because it basically describes the standard of care imposed upon children rather than professionals (though substituting "education" for "age"). Answer choice C is incorrect because, although it lists the correct standard for professionals, it fails to account for a physician's duty to obtain informed consent. Answer choice D is incorrect because a physician is under a special duty to obtain informed consent, which the physician failed to do in this case.

What must be foreseeable to D - the act, or the injury?

Injury only, does not matter how unforeseeable the act is. i.e.

Assault RS

Intent Apprehension Imminent ASS The defendant must intend to cause the plaintiff's apprehension of an imminent harmful or offensive contact or intend to cause harmful or offensive bodily contact with the victim. An assault is the plaintiff's reasonable *apprehension* of an *imminent harm*ful or offensive bodily contact *caused* by the defendant's action or threat with the *intent* to cause either the apprehension of such contact or the contact itself. 1. Bodily Contact Bodily contact is not required for assault. The prototypical assault occurs when the plaintiff sees the defendant throw a punch at him. Regardless of whether the punch connects (and therefore causes a battery), the apprehension of the contact is sufficient for assault. 2. Reasonable Apprehension A plaintiff's apprehension must be reasonable. Unlike with battery, the plaintiff must be aware of or have knowledge of the defendant's act. The defendant's apparent ability to cause harm (e.g., a "real-looking" toy gun) can be sufficient to place the plaintiff in apprehension of harm. Even if the victim is confident that he can prevent the threatened harm, there may still be an assault. 3. Imminence The threatened bodily harm or offensive contact must be imminent, i.e., without significant delay. Threats of future harm are insufficient, as are threats made by a defendant too far away to inflict any harm. Example: If the defendant calls and threatens the plaintiff from across the city, then the threat is not imminent and therefore there is no assault. 4. Mere Words It is sometimes said that "mere words alone do not constitute an assault." However, words coupled with conduct or other circumstances may be sufficient. If the defendant is able to carry out the threat imminently and takes action designed to put the victim in a state of apprehension, then there may be an assault. Example: If the defendant sneaks up behind the plaintiff in a dark alley and utters in a menacing voice, "Your money or your life," then an assault may be complete. 5. Intent The defendant must intend to cause the plaintiff's apprehension of an imminent harmful or offensive contact or intend to cause harmful or offensive bodily contact with the victim. The defendant's own words, however, can negate the intent. Example: If the defendant says, "If you were not such a good friend, I would punch you," then there is no assault. The doctrine of transferred intent applies to assault. 6. Damages No proof of actual damages is required. The victim can recover nominal damages and, in appropriate cases, punitive damages. If the plaintiff sustains damages from physical harm, such as a heart attack resulting from the assault, then he may recover these as well.

Rule statement: Intrusion upon seclusion

Intruding Objectionable Seclusion/Private affairs The defendant's act of intruding, physically or otherwise, into the plaintiff's private affairs, solitude, or seclusion in a manner or to a degree *objectionable to a reasonable person* establishes liability. In this case, the defendant hacked into the plaintiff's personal email account. This intrusion would be considered objectionable to a reasonable person. Therefore, the plaintiff would likely recover under this doctrine. Note that the plaintiff need not prove special damages for intrusion upon seclusion. Emotional distress is sufficient. Answer choice A is incorrect because, although the defendant did threaten the plaintiff, there was no apprehension of an imminent contact (the defendant said "tomorrow"). Answer choice C is incorrect. The requirement of publicity was not satisfied by the defendant's actions. The requirement of publicity in the public disclosure tort requires far broader dissemination of the information than is required under the "publication" requirement of defamation. The information must be communicated at large or to so many people that it is substantially certain to become one of public knowledge. Here, the defendant only emailed the plaintiff's wife and his two closest advisors regarding the plaintiff's relations with the former staff member. Although arguable, it is unlikely that the relationship is substantially certain to become one of public knowledge based on the limited publication. Answer choice D is incorrect because the statement the defendant made to the plaintiff's wife and two closest advisors was true. Truth is a complete defense to defamation.

Is injury. To rescuers sufficiently. Foreseeable under. The Cardozo view?**

It is well established that injury to rescuers is sufficiently foreseeable to support proximate cause. As Cardozo said, "Danger invites rescue." Answer choice A is incorrect. Even though the chain of causation here is lengthy, it is well established that injury to rescuers is sufficiently foreseeable to support proximate cause. Answer choice B is incorrect. The danger to pedestrians of speeding cars on a highway is foreseeable. But the defense of assumption of risk requires more specific knowledge of the risk that is run, as well as either agreement to take responsibility for the risk or unreasonable risk-taking on the part of the plaintiff. There is no indication here that the truck driver agreed to take full responsibility for the risk of being struck by a speeding car or unreasonably exposed himself to that risk. Answer choice C is incorrect. In order to establish proximate cause, something more specific than "injuries" broadly defined must be foreseeable. The kind of injury that had to be foreseeable to the manufacturer in this case is that someone engaged in a rescue attempt after a car accident might be struck by a speeding vehicle on the road.

What is the Cardozo view?

It is well established that injury to rescuers is sufficiently foreseeable to support proximate cause. As Cardozo said, "Danger invites rescue." Answer choice A is incorrect. Even though the chain of causation here is lengthy, it is well established that injury to rescuers is sufficiently foreseeable to support proximate cause. Answer choice B is incorrect. The danger to pedestrians of speeding cars on a highway is foreseeable. But the defense of assumption of risk requires more specific knowledge of the risk that is run, as well as either agreement to take responsibility for the risk or unreasonable risk-taking on the part of the plaintiff. There is no indication here that the truck driver agreed to take full responsibility for the risk of being struck by a speeding car or unreasonably exposed himself to that risk. Answer choice C is incorrect. In order to establish proximate cause, something more specific than "injuries" broadly defined must be foreseeable. The kind of injury that had to be foreseeable to the manufacturer in this case is that someone engaged in a rescue attempt after a car accident might be struck by a speeding vehicle on the road.

What is a parent's liability with respect to their minor child?

KID^2 - Knowledge -- Intentional/Negligent harm -- Duty of care if control Parents are liable for their own negligence with respect to their minor child's conduct. A parent is under a duty to exercise reasonable care to prevent a minor child from intentionally or negligently harming a third party, provided the parent has the ability to control the child, and knows or should know of the necessity and opportunity for exercising such control. Here, despite knowing of the need for controlling the boy, the father failed to adequately supervise the boy's actions because he did not ensure that his son was swallowing the pills, and he was at the bar drinking instead of supervising his son at the bowling alley. Answer choice A is incorrect because although the father did not know the boy was spitting out his pills, he still should have known of the necessity to adequately supervise the boy at the bowling alley. Answer choice B is incorrect. The fact that the boy committed an intentional tort does not preclude the father from being held liable for failing to control the boy. A parent can be liable for failing to prevent a minor child from intentionally or negligently harming a third party. Answer choice C is incorrect because the father is liable for his own negligence, but he will not be vicariously liable for the boy's tortious conduct.

Rule statement: Misappropriation of the right to publicity

MISA: Misappropriate causing Injury Consent Advantage Taylor swift, all you are is MEAN Misappropriate Advantage Consent Injury To prove the misappropriation of the right to publicity, the plaintiff must prove that the defendant appropriated the plaintiff's name, likeness, or identity for the defendant's advantage without the plaintiff's consent, and that there was a resulting injury to the plaintiff. Here, the stunt double pretended to be the movie star without his consent. His actions injured the movie star's reputation, and as a result the stunt double was able to get a few acting roles. Therefore, the elements of the movie star's claim have been met.

In which doctrine will P liability completely bar recovery?

MOD = tightWAD 1) Pure comparative negligence In jurisdictions that have adopted the doctrine of pure comparative negligence, a plaintiff's contributory negligence is not a complete bar to recovery. Instead, the plaintiff's full damages are calculated by the trier of fact and then reduced by the proportion that the plaintiff's fault bears to the total harm (e.g., if the plaintiff's full damages are $100,000, the plaintiff is 80% at fault, and the defendant is 20% at fault, then the plaintiff will recover $20,000). EXAM NOTE: Always apply pure comparative negligence on the MBE unless the facts instruct you to apply a different test. 2) Modified or partial comparative fault A majority of comparative-fault jurisdictions apply modified comparative fault. In these jurisdictions: i) If the plaintiff is less at fault than the defendant, then the plaintiff's recovery is reduced by his percentage of fault, just as in a pure comparative-fault jurisdiction; ii) If the plaintiff is more at fault than the defendant, then the plaintiff's recovery is barred, just as in a contributory-negligence jurisdiction; iii) In the vast majority of modified comparative-fault jurisdictions, if the plaintiff and the defendant are found to be equally at fault, then the plaintiff recovers 50% of his total damages. In a few modified comparative-fault jurisdictions, the plaintiff recovers nothing when the jury finds that the plaintiff and the defendant are equally at fault.

Guest statute

Minority jurisdiction requires drivers to refrain from reckless or wanton disregard for passengers, rather than a standard of ordinary due care

Intentional misrepresentation: RS

Misrep Induce Scienter Reliance Economic MISREpresentation misrepresentation by the defendant, scienter, intent to induce the plaintiff's reliance, justifiable reliance by the plaintiff, and actual economic damages. To recover for intentional misrepresentation, the plaintiff must establish a misrepresentation by the defendant, scienter, intent to induce the plaintiff's reliance, justifiable reliance by the plaintiff, and actual economic damages. Here, the woman relied on the trainer's false assertions regarding the pills with the intent to induce her reliance on his statements, and the woman justifiably relied upon his false assertions. However, the woman did not suffer any pecuniary loss since she did not pay for the pills. Thus, while she likely has some tortious claim against the trainer, she will not succeed on a claim for intentional misrepresentation.

Negligent Misrep RS

Negligent Misrepresentation False / Negligent / business / reliance / K This tort is based upon a breach of the duty to supply correct information and often arises in the context of accountants and other suppliers of commercial information. 1. Elements and Scope In a majority of jurisdictions and under the Second Restatement, the elements are: i) The defendant provides false information; ii) As a result of the defendant's negligence; iii) During the course of his business or profession; iv) Causing the plaintiff to justifiably rely upon the information; and v) The plaintiff either is in a contractual relationship with the defendant or is a third party known by the defendant as one for whose benefit the information is supplied. Under this rule, the accountant who regularly conducts audits and furnishes financial statements and opinions routinely required by lenders, investors, purchasers, or others is not liable unless she is informed that an identified third party or identified third parties will be using the statement for a particular purpose. 2. Defenses Unlike in intentional misrepresentation, in negligent misrepresentation, negligence defenses can be raised. 3. Damages The plaintiff can recover reliance (out-of-pocket) damages, as well as any other consequential pecuniary damages, if negligent misrepresentation is proven with sufficient certainty. 4. Distinguished From Ordinary Negligence The ordinary rules of negligence apply when physical harm is a foreseeable result of a negligent misrepresentation. Example: A defendant air traffic controller is liable for ordinary negligence when he negligently gives the pilot of an airplane incorrect information about the plane's location and speed and, as a result, the passenger-parachutist jumps to his death in a lake instead of at the target airfield.

What defences are available in strict liability?

No contributory negligence. The plaintiff's *assumption of the risk* bars his recovery in a strict-liability action. A plaintiff is barred from recovery when he unreasonably and voluntarily encounters a known, specific risk of harm.

Does the duty of reasonable care apply to trespassers?

No, it expands beyond the scope of the duty of reasonable care, unless there is an ancillary doctrine (attractive nuisance, which preserves the reasonable care standard) or is there is a concealed danger.

Do insults, threats and. Indignities rise to the level of IIED?

No. A defendant is liable for intentionally or recklessly acting with extreme and outrageous conduct that causes the plaintiff severe emotional distress. In this case, the paralegal clearly suffered emotional distress, but yelling mere insults, threats, or indignities at someone does not rise to the level of extreme and outrageous conduct.

Is 'coming to the nuisance' generally a defence?

No. To recover for a private nuisance, the instructor must prove that the neighbor's conduct substantially and unreasonably interfered with her use and enjoyment of her property. A substantial interference is one that would be offensive, inconvenient, or annoying to a normal, reasonable person in the community. A person with special sensitivities can recover only if the average person would be offended, inconvenienced, or annoyed. Here, the instructor has special sensitivities due to the use of the property as a place to hold yoga and meditation classes. However, she cannot recover because an average person, such as the other neighbors, would not be annoyed or inconvenienced by the neighbor's conduct. Answer choice A is incorrect because it misstates the test, which is whether the interference would be unreasonable, and offensive, inconvenient, or annoying to a normal, average person in the community. Simply interfering with the yoga instructor's planned use of her property is not sufficient to succeed on this claim. Answer choice B is incorrect because there is no law limiting one's right to enjoy one's property to economically useful conduct. An interference is unreasonable if the injury caused by the defendant outweighs the usefulness of his actions, but it must also be substantial to warrant a claim for private nuisance. Answer choice C is incorrect because it is generally not a defense that the plaintiff "came to the nuisance" by purchasing property in the vicinity of the defendant's premises with knowledge of the alleged nuisance.

Public figure liability re: IIED

Normally, a defendant is liable for Intentionally or recklessly acting with Extreme and outrageous conduct that Causes the plaintiff severe emotional Distress. (sImone Intentionally Induces Extreme Distress) unless MALEFICENT Public figures and public officials may not recover for the tort of intentional infliction of emotional distress by reason of PUBLICATION without showing in addition that the publication contains a false statement of fact which was made with "actual MALICE," i.e., with knowledge that the statement was false or with reckless disregard as to whether or not it was true. In this case, the paparazzo will not be held liable as he did not publish a false statement of fact with "actual malice." Instead, he simply broadcast a video that showed the neighbor shooting the dog owner's dog. Answer choice A is incorrect because it is an incorrect statement of law, as physical harm need not be proven in a claim for intentional infliction of emotional distress. Answer choice C is incorrect because the dog owner must prove not only that she suffered severe emotional distress, but that the distress was the result of the paparazzo intentionally or recklessly acting with extreme and outrageous conduct. Answer choice D is incorrect because the conduct of the neighbor does not apply to the dog owner's claim against the paparazzo.

Under what conditions is an insane delusion a justifiable defence to a tort claim?*

One is liable for battery when she intentionally causes a harmful or offensive contact with the person of another and acts with the intent to cause such contact. In this case, because the woman thought that the guest was a lion, she will not be liable for battery because she was not aware that she was striking a person. Answer choice B is incorrect. A mentally ill person may be held liable for a battery if she has the mental capacity to form the intent to strike a person, even if she is unable to understand the wrongfulness of her act. Therefore, this is not an effective defense. Answer choice C is incorrect because there is no requirement that the woman intended to cause harm to the guest, only that she intended to cause the unpermitted contact. Answer choice D is incorrect. The woman believed that the guest was about to attack her. Thus, she would likely be permitted to use reasonable force to defend herself. However, a justified reason or excuse, even if successful, is not as good of a defense as the failure to establish a required element for battery. Therefore, answer choice A is better than answer choice D.

When is an emergency professional permitted to recover damages?

Only barred if inherent in the job. An emergency professional, such as a police officer or firefighter, is barred from recovering damages from the party whose negligence caused the professional's injury if the injury results from a risk inherent in the job ("firefighter's rule"). In this case, the firefighter's injury was caused by the owner's failure to properly maintain the stairs leading into the apartment building. This injury did not result from a risk inherent in the job and consequently that firefighter can recover from the owner. The firefighter's rule would, however prevent the firefighter from recovering from the tenant whose negligence caused the fire which, in turn, led to the firefighter's presence in the apartment building where the firefighter was injured.

How are special sensitivities handled in nuisance actions?

Opposite of thin skull. A private nuisance is a thing or activity that substantially and unreasonably interferes with another individual's use or enjoyment of his land. A substantial interference is one that would be offensive, inconvenient, or annoying to a normal, reasonable person in the community. A person with special sensitivities can recover only if the average person would be offended, inconvenienced, or annoyed. In this case, only persons with rare olfactory conditions could smell the otherwise harmless gas emitted by the plant. Because the gas emissions would not be offensive to a normal, reasonable person in the community, the interference is not substantial and thus the man's action will fail.

Res ipsa loquitur elements

PRAYS - PRIEST BURGER Burger with shard of glass The thing speaks for itself (would not happen except in the presence of negligence) P not at fault In control of D

How are pure comparative damages calculated?

PU CO "Plaintiff's 'pooh pooh' cut' In jurisdictions that have adopted the doctrine of pure comparative negligence, a plaintiff's contributory negligence is not a complete bar to recovery. Instead, the plaintiff's full damages are calculated by the trier of fact and then reduced by the proportion that the plaintiff's fault bears to the total harm. Here, the plaintiff can recover $40,000. (The total damages of $100,000 are reduced by 60% of $100,000 "" the proportion that the plaintiff's fault bears to the total harm.)

What must be established in a negligence action?

Physicians are under a specific obligation to explain the risks of a medical procedure to a patient in advance of a patient's decision to consent to treatment. Failure to comply with this "informed consent" doctrine constitutes a breach of the physician's duty owed to the patient and is actionable as medical malpractice (medical negligence). As with any negligence action, the plaintiff must establish the following four elements: duty, breach, causation, and damages. Here, the woman did not suffer any damages resulting from the plastic surgeon's failure to disclose the risk of numbness when undergoing liposuction. Absent injury, the woman cannot recover in a negligence action.

When P is a minor, what should we immediately do?

Place P's guardian in his place for suit.

What defences should immediately come to mind when we encounter any torts that are even remotely public (i.e., NS Power)?

Privilege Providing electrical power to the community is arguably an essential public service akin to construction of utility or sewer lines. As such, it likely exempts PLC from strict liability; however, liability may still exist under a negligence theory.

Define res ipsa loquitur (rule statement)

RIL: speaks for itself - Control, no negligence Under the doctrine of res ipsa loquitur, the trier of fact may infer the existence of the defendant's negligent conduct in the absence of direct evidence of such negligence. Res ipsa is *circumstantial evidence* of negligence that does not change the standard of care. To be able to invoke the doctrine, the plaintiff must establish that: (i) the accident was of a kind that ordinarily does not occur in the absence of negligence; (ii) it was caused by an agent or instrumentality within the exclusive control of the defendant; and (iii) it was not due to any action on the part of the plaintiff. In this case, a burger patty would not ordinarily have a glass shard embedded in it in the absence of negligence. The burger patty was under the exclusive control of the restaurant at the time of the incident, as the woman provided evidence that all of the burger patties were made in the restaurant's kitchen and only the restaurant's employees had access to this kitchen. Furthermore, the injury was not caused by any action by the woman. Thus, the woman has satisfied a prima facie case of res ipsa loquitur. Accordingly, the court should not grant the restaurant's motion for a directed verdict. Answer choice B is incorrect. The woman brought an action against the restaurant based on negligence, not strict liability. Thus, the possibility of the restaurant's liability arising under strict liability is not relevant.

What is an abnormally dangerous activity?

Risk of harm with care Uncommon Like dropping a bowling ball from an overpass. Abnormally dangerous means that an activity creates a foreseeable and highly significant risk of physical harm even when reasonable care is exercised, and the activity is not commonly engaged in. In this case, the use of the air machines does not create a foreseeable and highly significant risk of physical harm even when reasonable care is exercised. By performing daily inspections and gating the air machines, as well as posting warning signs, Jumping Land exercised reasonable care which minimized any foreseeable risk of physical harm. Thus, the air machines do not fit the definition of abnormally dangerous.

What are the two ways of assessing the standard of care owed?

SOC = Special? In most cases, the standard of care imposed is that of a reasonably prudent person as measured by an objective standard. However, when a *special relationship* exists between the plaintiff and defendant, the defendant may owe a different duty of care to the plaintiff. Here, PLC is an owner or occupier of land who may be liable for injuries to Peter, a child trespasser, under the attractive nuisance doctrine.

What is gthe extent of NR?

The elements of negligent misrepresentation are: (i) the defendant provided false information, (ii) as a result of the defendant's negligence, (iii) during the course of his business or profession, (iv) causing the plaintiff to justifiably rely upon the information, and (v) the plaintiff either is in a contractual relationship with the defendant or is a third party known by the defendant as one for whose benefit the information is supplied. Here, although the elements of negligent misrepresentation are otherwise met, the accountant is not liable if the third party's use of the information is of a different character than the use for which the accountant provided the information. That is the case here because the accountant's opinion was prepared to aid the supplier in determining whether to sell goods to the retail company rather than to purchase the company. Answer choice A is incorrect because, while the supplier's reliance on the accountant's opinion is necessary to establish negligent misrepresentation, it alone is not sufficient for the reason discussed with respect to answer choice D. Answer choice B is incorrect because, even though the accountant was aware that the supplier was the intended user of the opinion, the answer choice does not allow for the fact that liability is nevertheless limited. Answer choice C is incorrect because the action is for negligent misrepresentation, not intentional misrepresentation. Consequently, the accountant's negligence can serve as a basis for liability.

What degree of force is permissible against. A trespasser?

The farmer was privileged to use reasonable force to prevent or end a trespasser's intrusion upon his land or to protect his property, but he was not privileged to use force that threatened serious bodily injury unless he was himself in danger of serious bodily harm. The force the farmer used was sufficient to and did in fact cause serious bodily injury. The child appeared to pose no threat of bodily harm to the farmer and could have been deterred by less forceful means. Answer choice A is incorrect. The child can state a claim for battery because there was an intentional infliction of a harmful contact. To support a battery action, the contact need not involve force so great as to threaten death. Answer choice B is incorrect because even if the child had been a thief, the privilege to use reasonable force to protect one's property does not extend to the use of force likely to cause serious bodily harm when there is no threat of such harm to oneself. As previously stated, the child appeared to pose no threat of bodily harm to the farmer and could have been deterred by less forceful means. Answer choice C is incorrect. The farmer was not required to have posted a warning in order to have had a privilege to protect his property by the use of reasonable force, although the absence of a warning sign may become a factor in determining whether the steps he took were in fact reasonable. In evaluating whether his actions were reasonable as a defense of his property, the court will ask whether the force he used was excessive. Because the force the farmer used was greater than necessary and was intended to cause serious bodily harm, it was excessive as a defense of property.

When are attorney's fees recoverable in. A personal injury action??

The measure of damages in a personal injury action includes all actual damages incurred, past and future pain and suffering (e.g., emotional distress), medical expenses, lost wages and any reduction in future earnings capacity, and loss of consortium. Under the "thin-skull" or "eggshell-plaintiff" rule, the defendant is liable for the full extent of the plaintiff's injuries due to the plaintiff's pre-existing physical or mental condition or vulnerability, even if the extent is unusual or unforeseeable. Attorney's fees in a personal injury suit are not recoverable. In this case, the plaintiff may not obtain attorneys' fees. Answer choice B is incorrect because the fact that the worker's preexisting asthma exacerbated the damage to her airways is irrelevant, as a defendant is liable for the full extent of the plaintiff's injuries, even if increased by the plaintiff's preexisting physical or mental condition. Answer choices C and D are incorrect because both future medical expenses and pain and suffering damages are recoverable in a personal injury negligence action.

Name and describe the two subtests for causation. Under the first, what is the threshold that must be met? Under the second, what is a potential excuse from liability?

The plaintiff must prove that the defendant's actions were both the actual cause (cause-in-fact) and proximate (legal) cause of the plaintiff's injury. Actual Cause If the plaintiff's injury would not have occurred but for the defendant's acts, then the defendant's conduct is the cause-in-fact of the harm. If there is more than one cause-in-fact, the defendant is the actual cause of the injury when the defendant's conduct was a substantial factor in causing the damages. Here, Peter's injury would not have occurred but for PLC's operation of the high-voltage substation in a field adjacent to the residential area. Arguably, his own act of chasing the bird into the substation is also the "but for" cause of his injuries. However, since PLC's operation of the substation was a substantial factor in causing Peter's injury, PLC will not escape liability simply because Peter's own actions contributed to his damages. Proximate Cause To prove proximate cause, the plaintiff must show that his injuries were the foreseeable result of the defendant's conduct. As discussed above, PLC could have foreseen that a child would sneak into the substation area and suffer an injury while climbing the barbed wire fence or substation. On the other hand, PLC may not have anticipated that a child would be capable of climbing a six-foot high fence topped with barbed wire. Further, it is even less likely that PLC could have anticipated that a child would then scale a steel support to catch a bird and get burned by the high voltage wire in the process of doing so. However, Father need only prove that it was foreseeable to PLC that if someone were to enter the fenced-in area, he or she could be harmed by electrocution or burning, regardless of how the actual injury came about. PLC may argue that Peter's act of chasing the bird was an unforeseeable intervening cause. However, a defendant will still be liable for its own acts if the result was nonetheless foreseeable. Here, it was foreseeable that a child would climb onto the substation and suffer injury from contact with the high voltage wire. Thus, PLC's operation of the substation was the proximate cause of Peter's injury.

Who is liable to a plaintiff for a defective product?

The plumbing store is liable for the harm caused to the plaintiff by the defective product (i.e., a hot water heater). To be subject to strict liability for a defective product, the defendant must be in the business of selling or otherwise distributing products of the type that harmed the plaintiff. Included as a seller are the manufacturer of the product, its distributor, and its retail seller. As long as the seller is a commercial supplier of the product, the seller is subject to strict liability for a defective product even if the seller was not responsible for the defect in any way. Here, the product is defective due to its design; the walls of the tank were too thin. Although the local plumbing store did not manufacture the defective hot water heater, the store, as a commercial supplier of the product, is strictly liable for it. However, the supplier of the plastic is not liable. The commercial supplier of a component that is integrated into a product during its manufacture is not liable unless the component itself is defective or the supplier substantially participates in the integration process and the integration of the component causes the product to be defective. Here, the plastic resin provided by the supplier was not itself defective, and, although the supplier participated in the integration process, the supplier's participation did not lead to the defective hot water heater. Had the supplier's advice been followed, the hot water heater tank would not have melted.

Wild anima liability - what is An important issue to consider?

The possessor of a wild animal is strictly liable for harm done by that animal, in spite of any precautions the possessor has taken to confine the animal or prevent the harm, if the harm arises from a dangerous propensity that is characteristic of such a wild animal or of which the owner has reason to know. Strict liability also applies to an injury caused by a plaintiff's fearful reaction to the sight of an unrestrained wild animal. In this case, although the defendant owned a wild animal, she is not strictly liable for the injury caused by the plaintiff's fearful reaction to the sight of the alligator because the alligator was not roaming free, but instead was confined within a pen. Consequently, the defendant is not liable to the plaintiff in negligence, nor is she strictly liable, because she exercised reasonable care in confining the alligator. Answer B is incorrect because strict liability for harm caused by wild animals extends to harm caused by the reasonable fear people feel in the vicinity of the wild animal, if unrestrained. Answer choice C is incorrect because compliance with a statute, a regulation, or an ordinance generally does not prove the absence of negligence, and would not protect the defendant from strict liability if the animal had escaped. Answer choice D is incorrect because the injury need not be caused by the alligator's dangerous propensities; an injury caused by the plaintiff's fearful reaction to an unrestrained wild animal is sufficient.

IIED: Rule Statement

The tort of intentional infliction of emotional distress has four elements: (1) the defendant must act intentionally or recklessly; (2) the defendant's conduct must be extreme and outrageous; and (3) the conduct must be the cause (4) of severe emotional distress.

What role does consent play in claims of battery and assault?***

There is no battery if the plaintiff consented to the act, either expressly or by virtue of participating in a particular event or situation. In this case, the facts do not indicate that the plaintiff expressly consented to the defendant's conduct. Similarly, the plaintiff did not consent to having objects thrown at him while in class merely by being present in the classroom. Answer choice A is incorrect because although the school rules would be relevant, they would be relevant only as evidence of whether the plaintiff did or did not implicitly consent. Answer choice B is incorrect because the tort of battery does not require a showing that the defendant intended to cause physical injury. Answer choice D is incorrect because the tort of battery entails liability for any harm even if the harm is worse than anticipated because of the weakness or clumsiness of the plaintiff.

When is private nuisance justifiable?

To establish a private nuisance, the inference with the use or enjoyment of another individual's property must be substantial and unreasonable. The interference is deemed unreasonable if the injury caused by the defendant outweighs the usefulness of his actions. Here, if the buyer aquired the dogs for the purpose of protection, the utility of the dogs to the defendant would be bolstered. Answer choice A is incorrect because anyone with possessory rights in real property who is allegedly subject to a private nuisance may bring an action. Because the chef, as a tenant, is entitled to possession of the property, he may pursue a lawsuit. Answer choice B is incorrect because a person may seek purely economic damages in a private nuisance action; physical injury is not required. Answer choice C is incorrect because, while it may be advantageous for a person who experiences a private nuisance to contact the perpetrator of the nuisance before filing suit to resolve the problem, there is no legal requirement that he do so.

What threshold can we expect manufacturers to adhere to in their duty to provide a warning?

To prevail on a claim under a strict products liability theory for failure to warn, a plaintiff must prove that the manufacturer failed to provide an adequate warning related to the risks of using the product. A failure to warn defect exists if there were foreseeable risks of harm, not obvious to an ordinary user of the product, that could have been reduced or avoided by providing reasonable instructions or warnings. The failure to include the instructions or warnings prevents the product from being reasonably safe. Here, there was a foreseeable risk of the seat collapsing, and this would not be obvious to an ordinary passenger flying on an airplane. The failure to include a maximum weight warning rendered the airplane seat not reasonably safe.

How do zoning requirements impact a. Nuisance claim?? What is the rule statement for nuisance (private)?

To prevail on a nuisance claim, the plaintiff must show that the defendant's activity has substantially and unreasonably interfered with the plaintiff's use and enjoyment of his property. The evidence that there has been a significant decrease in the value of the landowner's property and that his business is suffering a marked decline would support a claim that the intrusion by the gas company is both substantial and unreasonable. Answer choice A is incorrect because a showing of negligent behavior on the part of the defendant is not required to prevail on a nuisance claim. Answer choice B is incorrect. The character of the neighborhood may be considered in evaluating whether a property owner has created a nuisance, but compliance with zoning requirements does not defeat a nuisance claim if the defendant's activity substantially and unreasonably interferes with the landowner's use and enjoyment of his property, which appears to have happened in this case. Answer choice D is incorrect because a mere decline in property value is not enough to establish a nuisance claim.

Requirements for intentional interference with a contract

To prove intentional interference with a contract, the competitor must prove that: (1) a valid contract existed between the competitor and the manufacturer, (2) the company knew of the contractual relationship, (3) the company intentionally interfered with the contract, causing a breach, and (4) the breach caused damages to the competitor. Here, the company did not know of the contractual relationship between the competitor and the manufacturer, so it is not liable on these grounds. Answer choice A is incorrect because, even though a third party does enjoy a privilege to compete, such privilege is not absolute. Answer choice C is incorrect because, although a third party may not rely on a contractual defense, such as a violation of the Statute of Frauds, in defending an intentional interference with a contract action against it by a party to the contract, the third party must have knowledge of the contract at the time that it contacts a party to an existing contract. Answer choice D is incorrect because, although inducing a party to a contract to breach that contract is the primary means by which intentional tortious interference with a contract can be committed, the third party's actions must not only induce the breach but also be done with knowledge of the existence of such a contract.

What are the requirements for negligence per se?

To prove negligence per se, there must be a regulatory statute or ordinance that the defendant violates by failing to perform that duty. Additionally, the plaintiff must be in the class of people the statute intends to protect, the harm must be of the type the statute was intended to protect against, and the plaintiff's injuries must be proximately caused by the defendant's violation of the statute or ordinance. Here, the camp counselor used soy sauce at the camp in violation of the ordinance. However, the daughter suffered a bacterial infection from the soy sauce rather than an allergic reaction. Therefore, the harm she suffered was not of the type that the ordinance was intended to protect against.

Can additional damages be looped into strict liability?

To recover under a strict products liability theory, the plaintiff must plead and prove that the product was defective, the defect existed when it left the defendant's control, and the defect caused the plaintiff's injuries when used in an intended or reasonably foreseeable way. Here, the facts state that the table malfunctioned due to a manufacturing defect. Thus, it can be presumed that the defect existed when it left the manufacturer's control. The table was also being used for its intended purpose when the injury occurred. Therefore, the chiropractor will prevail under a strict products liability theory. Answer choice A is incorrect because it refers to the legal standard for negligent infliction of emotional distress. Answer choice C is incorrect because anyone foreseeably injured by a defective product or whose property is harmed by the product may bring a strict-liability action. Appropriate plaintiffs include purchasers, other users of the product, and even bystanders who suffer personal injury or property damage. Answer choice D is incorrect because it is the product that must be used in an intended or reasonably foreseeable way. Moreover, it was likely reasonably foreseeable in this situation that the chiropractor would be injured in such a way.

Trespass to Chattels vs. Conversion RS

Trespass to Chattels 1. Definition A defendant is liable for trespass to chattels (i.e., tangible personal property) if he intentionally interferes with the plaintiff's right of possession by either: i) Dispossessing the plaintiff of the chattel; or ii) Using or intermeddling with the plaintiff's chattel. Trespass to chattels requires that the plaintiff show actual harm to or deprivation of the use of the chattel for a substantial time. 2. Intent Only the intent to do the interfering act is necessary; the defendant need not have intended to interfere with another's possession of tangible property. The doctrine of transferred intent applies to trespass to chattels. 3. Appropriate Plaintiffs An action for trespass to chattels may be brought by anyone with possession or the immediate right to possession of the chattel. 4. Mistake Mistake by the defendant about the legality of his actions is not a defense. 5. Damages In a case of dispossession, a plaintiff may recover for: i) The actual damages caused by the interference; and ii) The loss of use. In circumstances of use or intermeddling, the plaintiff may recover only when there are actual damages. 6. Remedy The plaintiff may be entitled to compensation for the diminution in value or the cost of repair. Conversion 1. Definition A defendant is liable for conversion if he intentionally commits an act depriving the plaintiff of possession of her chattel or interfering with the plaintiff's chattel in a manner so serious as to deprive the plaintiff of the use of the chattel. The plaintiff's damages are the chattel's full value at the time of the conversion. Only personal property and intangibles that have been reduced to physical form (e.g., a promissory note) can be converted. 2. Intent The defendant must only intend to commit the act that interferes; intent to cause damage is not necessary. Mistake of law or fact is no defense (e.g., a purchaser of stolen goods is liable to the rightful owner). Accidentally damaging the plaintiff's chattel is not conversion if the defendant had permission to use the property. 3. Interference The defendant interferes with the plaintiff's chattel by exercising dominion or control over it. Examples of acts of conversion include wrongfully acquiring, transferring, or detaining; substantially changing; severely damaging or destroying; or misusing the chattel. Note that if the original acquisition of the chattel was not wrongful, then the plaintiff must demand the return of the chattel before she sues for conversion. 4. Distinguishing Conversion From Trespass to Chattels There is no specific rule as to what behavior constitutes conversion, as opposed to trespass to chattels; it is a matter of degree of seriousness. The following factors are considered: i) The duration and extent of the interference; ii) The defendant's intent to assert a right inconsistent with the rightful possessor; iii) The defendant's lack ofgood faith; iv) The expense or inconvenience to the plaintiff; and v) The extent of the harm to the chattel. Generally, the greater the degree of these factors, the greater the likelihood that a conversion has occurred. Conversion is an exercise of dominion or control over the plaintiff's personal property such that the court is justified in requiring the defendant to pay the plaintiff the full value of the property. Example: If an embittered defendant steals his ex-girlfriend's car and drives it into a lake, that is conversion. If he merely hits the hood of her car once with a hammer, that is trespass to chattels. 5. Damages The plaintiff may recover damages in the amount of the full value of the converted property at the time of the conversion. Alternatively, the plaintiff may bring an action for replevin to recover the chattel.

Trespass to Land RS and Defences

Trespass to Land: DIP IN on someone's land Damages Intent proper Plaintiff Invasion not Nuisance Intent / Invasion / Plaintiff / Nuisance / Damages Private or Public Necessity Trespass to land occurs when the defendant's intentional act causes a physical invasion of the land of another. 1. Intent The defendant need only have the intent to enter the land (or to cause a physical invasion), not the intent to commit a wrongful trespass. In other words, the defendant need not know that the land belongs to another. Mistake of fact is not a defense. Example: Assume that an erroneous survey of the defendant's property leads her to believe that an annoying cherry tree is on her property when in fact it is on her neighbor's property. She intentionally enters the space where the tree is located and cuts it down. Even though she reasonably believed that the tree was on her property, she still will be liable for trespass. The doctrine of transferred intent applies to trespass to land. 2. Physical Invasion The defendant need not personally enter onto the plaintiff's land; intentionally flooding the plaintiff's land, throwing rocks onto it, or intentionally emitting particulates into the air over the land will each suffice. Additionally, the defendant's failure to leave the plaintiff's property after his lawful right of entry has expired constitutes a physical invasion. A trespass may be committed on, above, or below the surface of the plaintiff's land. 3. Appropriate Plaintiffs Because it is the right to possession that is being protected, anyone in actual or constructive possession of land may bring an action for trespass (e.g., owner, lessee, adverse possessor). 4. Distinguished From Nuisance Trespass always requires an invasion or intrusion of land; nuisance may or may not involve intrusion. Trespass protects the possessor's interests in the land; nuisance protects the use and enjoyment of land. See also § III.D. Nuisance, infra. If no physical object enters onto the plaintiff's land (e.g., the defendant's floodlights project onto the plaintiff's land, or damage results from the defendant's blasting), then the case is generally treated as a nuisance or strict liability action (discussed in §§ III.D. Nuisance and V. Strict Liability, infra). 5. Damages No proof of actual damages is required. 6. Necessity as a Defense The privilege of necessity is available to a person who enters onto the land of another (or interferes with another's personal property) in order to prevent injury that is substantially more serious than the invasion or interference itself. The privilege of necessity applies only to intentional torts to property, including trespass to land, trespass to chattels, and conversion. a. Private necessity Private necessity is a qualified privilege to protect an interest of the defendant or a limited number of people. The privilege applies if the interference was reasonably necessary to prevent a threatened injury from nature or another force not connected with the plaintiff. The property owner is entitled to recover actual damages, but he cannot recover nominal or punitive damages nor can he use force to eject the defendant. Example: Assume that the defendant docks her valuable boat on the plaintiff property-owner's dock during a severe storm to prevent the destruction of the boat. During the storm, the winds knock the boat against the dock, causing damage to the dock. The defendant will be liable to the dock owner for the actual damages. EXAM NOTE: Private necessity, often referred to simply as "necessity," has been tested frequently on the MBE. b. Public necessity Under the doctrine of public necessity, private property may be intruded upon or destroyed when necessary to protect a large number of people from public calamities, such as the spread of a fire, the spread of a disease, or the advance of a hostile military force. The privilege is absolute. As long as the defendant acts reasonably, he is not liable for any damage to the property. He is not liable even if the original entry was not necessary, as long as he reasonably believed that the necessity existed. The privilege lasts only as long as the emergency continues. The privilege is available to private citizens or public officials, should the plaintiff seek to hold a public official personally liable.

What is a thin skull plaintiff?

Under the "thin-skull" or "eggshell-plaintiff" rule, the extent of the damages need never be foreseeable. The defendant is liable for the full extent of the plaintiff's injuries that may increase due to the plaintiff's preexisting physical or mental condition or vulnerability, even if the extent is unusual or unforeseeable. Here, the woman had a preexisting eye condition that resulted in temporary blindness when the employee sprayed her in the eyes with the perfume. Therefore, the employee will be liable for all of the woman's damages, even though the extent of her damages was unforeseeable. Answer choice A is incorrect because the extent of damages need not be foreseeable under the "eggshell-plaintiff" rule. Answer choice B is incorrect. Proximate cause requires that the plaintiff suffer a foreseeable harm that is not too remote, and is within the risk created by the defendant's conduct. Experiencing a significant eye issue due to perfume being sprayed into one's eyes is not so remote that the employee's conduct would not be the proximate cause of the woman's injuries. Answer choice C is incorrect because even though the employee must take the woman as she finds her, the employee would not be responsible for damages for the full amount of the woman's preexisting eye condition. She would only be responsible for the damages caused by her own negligence and for the aggravation to the woman's preexisting eye condition.

What is joint and several liability, and when does it apply?

Under the doctrine of joint and several liability (which you should assume applies on the bar exam, unless you are told otherwise), each of two or more tortfeasors who is found liable for a single and indivisible harm to the plaintiff is subject to liability to the plaintiff for the entire harm. The plaintiff has the choice of collecting the entire judgment from one defendant, the entire judgment from another defendant, or portions of the judgment from various defendants, as long as the plaintiff's entire recovery does not exceed the amount of the judgment. Here, the combined negligence of the three parents caused the child's indivisible injury, making each parent jointly and severally liable for the entire amount of the medical expenses. The mother (on behalf of her child) had the right to choose which parent to sue and from whom to recover. In turn, the third parent may sue the other two parents for contribution.

When can a dog bite case be dismissed?

Under the doctrine of res ipsa loquitur, the trier of fact may infer the existence of negligence in the absence of direct evidence of such negligence. Traditionally, for the doctrine of res ipsa loquitur to apply, the plaintiff must prove that: i) the accident is of the kind that would not ordinarily occur in the absence of negligence, ii) it was caused by an instrumentality within the exclusive control of the defendant, and iii) it was not due to any action of the plaintiff. To avoid a directed verdict, the plaintiff need only establish an inference of negligence. In this case, the woman would easily be able to prove the first and third elements of res ipsa loquitur, and would therefore only need to show that the dogs were in the exclusive control of the owner. While the dogs were not in the control of the man at the time that they bit the woman (as they had escaped), they were in his control when they escaped due to the unlocked cages. Therefore, although it remains to be seen whether the woman will be ultimately successful in her claim, she will likely survive a motion for a directed verdict. Answer choice A is incorrect because the doctrine of res ipsa loquitur exists specifically to infer the existence of negligence when no direct evidence exists.

What is the theory of alternative causation, and how is it applied?

Under the theory of alternative causation, if the plaintiff's harm was caused by a small number of defendants (usually between two and five), all of those defendants acted tortiously, and all of them are before the court, the court may shift the burden of proof to the defendants to prove that his conduct was not the cause of the plaintiff's harm. However, the initial burden is on the plaintiff is to prove that all of the defendants did, in fact, act negligently. In this case, while the chef can prove that someone acted negligently by subjecting the wine to an unacceptably high temperature, he cannot prove that both the collector and the delivery driver actually acted negligently. Accordingly, the chef has not established his prima facie case for negligence. Answer choice A is incorrect because joint and several liability applies when the tortious acts of two or more tortfeasors combine to produce an indivisible harm. Here, the chef has not presented evidence that both parties committed tortious acts. The chef must first establish the inference of negligence by both parties before this is an effective argument. Answer choice B is incorrect. Res ipsa loquitor can be used against multiple defendants if the plaintiff is unable to prove who among them was negligent, but this is generally used against parties who are at least acting in concert, such as in a surgical setting in a medical malpractice action, or occasionally in products liability actions. Typically, to hold two or more defendants responsible for one harm, there must be some evidence of negligence on the part of both defendants.

What is the risk-utility defence to strict liability?

Using the risk-utility test, to prevail on a claim under a strict products liability design defect theory, a jury must determine whether the risks posed by a product outweigh its benefits. To succeed, a plaintiff must prove that a reasonable alternative design was available to the defendant and the failure to use that design has rendered the product not reasonably safe. The reasonable alternative design must be economically feasible. Here, if the finder of fact determined that there was no economically feasible alternative design, the plaintiff could not prevail under a strict liability theory. Answer choice B is incorrect because the commercial manufacturer, distributor, retailer, or seller of a product owes a duty of reasonable care to any foreseeable plaintiff; it is not necessary that the plaintiff be the purchaser of the defective product.

TO what extent does the risk utility test apply?

Using the risk-utility test, to prevail on a claim under a strict-products-liability-design-defect theory, the jury must determine whether the risks posed by the product outweigh its benefits. A plaintiff generally must prove that a reasonable alternative design was available to the defendant and the failure to use that design has rendered the product not reasonably safe. The alternative design must be economically feasible. Here, the manufacturer could have made an engine that could combust untreated vegetable oil, but it would not have been economically feasible. Accordingly, the environmentalist's claim will not succeed. Answer choice A is incorrect because foreseeable misuse, alteration, or modification usually does not preclude recovery. Here, the environmentalist's misuse of the vehicle's engine by failing to use the chemical additives was foreseeable enough to warrant a manufacturer's warning and detailed instructions. Therefore, this form of misuse was likely foreseeable, and although it may have reduced any damage award, it is unlikely to bar the environmentalist's recovery.

When is emergency professional barred from recovering?

When risk is inherent in job. An emergency professional, such as a police officer or firefighter, is barred from recovering damages from the party whose negligence caused the professional's injury if the injury results from a risk inherent in the job ("firefighter's rule"). Here, the police officer's injury resulted from a risk inherent in the job. The driver's abrupt swerving of his truck in response to the police officer's presence was a risk involved in a police pursuit/investigation. Therefore, the injuries sustained by the police officer in the crash as a result of the driver's actions were inherent to his job and, consequently, the police officer cannot recover from the driver for his injuries.

Party's financial resources and impact on case???

Whether the patient has the resources to satisfy an adverse judgment is irrelevant to the judgment itself and should not be a subject of argument on the issue of liability in the case, although the patient's financial situation might affect a lawyer's decision to take on the case. Answer choice A is incorrect. The nurse's professional role is relevant because it might support a defense based on assumption of risk. Answer choice B is incorrect. The nurse's specialty training is relevant because it might support a defense based on either assumption of risk or contributory negligence. The patient could argue that a professional with the nurse's training should have foreseen the risk of this happening and taken steps to protect herself. Answer choice C is incorrect. The patient's mental state is relevant because the patient's actions were no doubt caused by the mental illness and thus fell within the risks likely assumed and anticipated by the nurse.

Can a landowner seek damages for a trespass due to private necessity?

Yes. Trespass to land occurs when the defendant's intentional act causes a physical invasion of the land of another. Private necessity is a qualified privilege that applies if the interference was reasonably necessary to prevent a threatened injury from nature or another force not connected with the plaintiff. However, even when this privilege applies, the property owner is entitled to recover actual damages. Here, the privilege of private necessity applies, but the rancher will still be able to recover for the actual damage done to his fence.

In which doctrine does last clear chance apply?

You CONTRIBUTED. (not comparative

What is a valid defence to a strict liability action?

assumption of the risk can serve as a defense to a strict liability action. The neighbor was aware of the cat's dangerous propensities and nevertheless voluntarily attempted to capture it. Answer choice A is incorrect because a domestic animal's owner may be strictly liable for harm caused by the animal when he knows or has reason to know of the animal's dangerous propensities and the harm results from those dangerous propensities. If the owner is aware of the animal's dangerous propensities, the owner may be strictly liable even though the animal has not previously injured anyone. Answer choice C is incorrect because the liability of an owner of a domestic animal for harm caused by the animal is not limited to negligence, but may also be based on strict liability. Answer choice D is incorrect. The owner of any animal, wild or domestic (other than household pets) is strictly liable for any reasonably foreseeable damage caused by his animal while trespassing on the land of another. However, the exception for household pets does not apply if the owner knows or has reason to know that the dog or cat is intruding on another's property in a way that has a tendency to cause substantial harm. Here, the exception for household pets does not apply because the homeowner knew of his cat's dangerous propensities, which could (and did) cause substantial harm.

Private Nuisance Rule Statement

nUISance Substantially Interferes Use Enjoyment Unless CC (Coming or Compliance) 1. Private Nuisance a. Definition A private nuisance is a thing or activity that substantially and unreasonably interferes with another individual's use or enjoyment of his land. b. Nature of the defendant's conduct The interference must be intentional, negligent, reckless, or the result of abnormally dangerous conduct to constitute nuisance. c. Appropriate plaintiffs Anyone with possessory rights in real property may bring a nuisance claim. d. Substantial interference A substantial interference is one that would be offensive, inconvenient, or annoying to a normal, reasonable person in the community. A person with special sensitivities can recover only if the average person would be offended, inconvenienced, or annoyed. Conversely, a "thick-skinned" plaintiff who is not offended, inconvenienced, or annoyed is nevertheless entitled to recover if an average reasonable person would be, although the amount of damages may be affected. e. Unreasonable interference The interference is unreasonable if the injury caused by the defendant outweighs the usefulness of his actions. f. Distinguished from trespass 1) Physical invasion Trespass requires a physical invasion of the plaintiff's property. Nuisance does not require physical invasion, but physical invasion may constitute a nuisance. Example: If the defendant's factory emits particulates that settle on the plaintiff's property, then the defendant may be liable for both trespass and private nuisance. 2) Substantial interference Private nuisance requires substantial interference with the plaintiff's use and enjoyment of her property. Trespass, however, does not require a substantial intrusion. Example: A defendant's merely walking onto the plaintiff's land, if unprivileged and not consented to, is a trespass. 3) Duration Generally, a nuisance is continuous. A trespass may be a one-time event, episodic, or continuous. g. Access to light Historically, courts have refused to find the obstruction of sunlight as creating a private nuisance. h. Defenses to private nuisance Apart from challenging the elements of nuisance, the defenses available to a defendant turn on whether the defendant's conduct is intentional, negligent, or abnormally dangerous. For example, the plaintiff's negligence or assumption of the risk may be a defense to a nuisance (or reduce recovery in a comparative-fault jurisdiction) resulting from the defendant's negligence. 1) Regulatory compliance The fact that a defendant's behavior is in compliance with a statute, local ordinance, or administrative regulation is not a complete defense to a nuisance action. However, such statutory or regulatory compliance may be admitted as evidence as to whether the interference with the plaintiff's use and enjoyment of her land is unreasonable. For example, zoning regulations are typically regarded as admissible evidence in actions for nuisance, but they are not determinative. 2) Coming to the nuisance It is generally not a defense that the plaintiff "came to the nuisance" by purchasing property in the vicinity of the defendant's premises with knowledge of the nuisance operated by the defendant. However, the fact that the plaintiff moved to the nuisance is not irrelevant; it may be considered by the jury in determining whether the plaintiff can recover for the nuisance. In other words, the plaintiff's coming to the nuisance does not entitle the defendant to judgment as a matter of law, but it is evidence that the jury may consider. Conversely, ownership of land prior to the defendant's entry into the neighborhood will not, by itself, make the defendant's action a nuisance. The test is whether the defendant's action is unreasonable.

What are the elements of IIED, and what proximity is required?

sImone Intends to Induce Extreme Distress A defendant is liable for intentionally or recklessly acting with extreme or outrageous conduct that causes the plaintiff severe emotional distress. Here, a previous court must have found that the woman's behavior was outrageous because it granted a restraining order prohibiting any conduct with the man. Thus, for the wife to succeed in her suit as well, the extreme and outrageous conduct does not have to be specifically directed towards her. If a plaintiff's family suffers emotional distress because of a defendant's intentional infliction of emotional distress upon the plaintiff, the plaintiff's family can also sue for emotional distress if the family member contemporaneously perceived the defendant's conduct. For this reason, as an immediate family member of the married man who was present for the harassment and suffered her own emotional distress, the wife can also successfully sue the woman for intentional infliction of emotional distress. Answer choice B is incorrect because it states the rule for negligent infliction of emotional distress. Therefore, this reasoning would not support the wife's action for intentional infliction of emotional distress. Answer choice C is incorrect. Transferred intent may apply to intentional infliction of emotional distress if, instead of harming the intended person, the defendant's extreme conduct harms another. In this case, regardless of whether transferred intent applies, because the woman knew that the couple still lived together and that they were both experiencing her harassment, the wife could also sue the woman for intentional infliction of emotional distress as a member of the married man's immediate family who contemporaneously perceived the defendant's conduct. Answer choice D is incorrect because a previous court has already found that the woman's conduct was extreme or outrageous in the man's action against the woman. Therefore, this is an incorrect statement given the facts provided.

Is an employer liable for abnormally dangerous activity of employees, and on what basis?

t. While those who hire independent contractors are generally not vicariously liable for the torts of the independent contractors, a person who hires an independent contractor remains vicariously liable for certain conduct, including inherently dangerous (i.e., ultrahazardous) activities. In the applicable jurisdiction, fumigation is such a task. Answer choice B is incorrect because, although assumption of the risk is a defense to strict liability, the tenant did not voluntarily remain in his apartment. Merely having knowledge of the need to vacate the apartment does not constitute assumption of the risk.


Conjuntos de estudio relacionados

Lesson 3.1 Death Defined - Embalming 1

View Set

College Board: Unit 1 progress Check: MCQ

View Set

(PrepU) Chapter 43: Loss, Grief, and Dying

View Set

Biology Ch. 14 and 15 Test Topics

View Set

New Zealand CPL Aircraft technical knowledge and loading

View Set

bstrandable NCLEX Cardiovascular/Hematologic 1 of 2

View Set